[Boards: 3 / a / aco / adv / an / asp / b / bant / biz / c / can / cgl / ck / cm / co / cock / d / diy / e / fa / fap / fit / fitlit / g / gd / gif / h / hc / his / hm / hr / i / ic / int / jp / k / lgbt / lit / m / mlp / mlpol / mo / mtv / mu / n / news / o / out / outsoc / p / po / pol / qa / qst / r / r9k / s / s4s / sci / soc / sp / spa / t / tg / toy / trash / trv / tv / u / v / vg / vint / vip / vp / vr / w / wg / wsg / wsr / x / y ] [Search | Free Show | Home]

ITT solve the math problem of the person above you and post a

This is a blue board which means that it's for everybody (Safe For Work content only). If you see any adult content, please report it.

Thread replies: 58
Thread images: 15

ITT solve the math problem of the person above you and post a harder problem.
I'll start with an easy one
>>
B
>>
>>9076883

B

Let [math]f^n(0)[/math] denote the nth iterate of [math]f[/math] applied to 0. Find a computable function [math]f[/math] such that the set [math]\{ f^n(0) : n \in {\bf N} \} [/math] is not computable.
>>
File: hmm.png (7KB, 837x318px) Image search: [Google]
hmm.png
7KB, 837x318px
>>9076883
legit not enough info. tell me the right answer and i'll tell you why it's wrong

now answer mine
>>
>>9076888
58. Im on mobile so I cant post another.
>>
>>9076888
indeterminate
>>
>>9076888
It's B. Now share your ideas why there's not sufficient information for that answer and I'll show you why there actually is.
>>
>>9076885
e
>>
>>9076899
Show work?
>>
>>9076902
This. Bonus points for showing handwritten work so no one cheats
>>
File: challenge_problem.jpg (30KB, 698x400px) Image search: [Google]
challenge_problem.jpg
30KB, 698x400px
>>9076888
The standard order of operations doesn't cover operations of the form a÷b(c), so there's no telling whether to multiply b and c first or to divide a by b first without defining the application of the order of operations in this case.

Pic related is the next harder challenge.
>>
File: c0rxM.jpg (24KB, 320x400px) Image search: [Google]
c0rxM.jpg
24KB, 320x400px
>>9076912
5, see pic related
>>
>>9076912
2+2=

>Find sin(arccos(4/5)) without using a calculator. Show your work.
>>
File: 1490081563163.png (135KB, 500x593px) Image search: [Google]
1490081563163.png
135KB, 500x593px
>>9076895
>Now share your ideas why there's not sufficient information for that answer and I'll show you why there actually is.
cos ur mom is a dumb whore lol
>>
>>9076912
stfu retard the whole point of posting that picture was to derail the thread not get some nerdy sperg to give me the right answer
>>
>>9076939
.6
>>
>>9076939
Guess 4chan doesn't render U+1F41F. That's what 2+2 equals.
>>
File: trig0.png (5KB, 479x366px) Image search: [Google]
trig0.png
5KB, 479x366px
>>9076939
I like this one. It took a second thought despite being 7th-grade trig.

By the Pythagorean theorem, the other side is 3, and since sine is the ratio of the opposite side to the hypotenuse, the answer is 3/5.
>>
It seems we need a new problem, so I'll offer one up. Prove a linear functional if continuous if and only if it has bounded norm.
>>
>>9076978


How many suns does it take to roll the moon. A square triangle is so very special mercedes
>>
>>9076991
Circumference of the sun/circumference of the moon
=
2,713,406 miles/6,786
=400
>>
>>9076885
the answer to OP's problem is E, brainlet.
>>
>>9077408
Taking the contrapositive of the statement "if he got all of them right, he got an A" yields "if he didn't get an A, he didn't get all of them right", yielding the answer B, you fucking brainlet
>>
>>9077423
the problem does not specify that he actually has to get all of them right to get an A. it specifies that if he gets all of them right then he gets an A, but it says nothing about if he got only 1 wrong, for example. he could still get an A. I reread it and I can't see a flaw with B, but E is more straight forward and correct.
>>
>>9077431
You're pantsu-on-head retarded mate.
>>
>>9077441
prove that E is wrong?
>>
File: judenmathematik.jpg (83KB, 320x400px) Image search: [Google]
judenmathematik.jpg
83KB, 320x400px
>>9076924
nice try you sneaky jew
>>
>>9077443
>prove that E is wrong
Where does it imply that you can't get an A without getting all of the multiple choice right?
>>
File: 1495056524894.png (11KB, 470x454px) Image search: [Google]
1495056524894.png
11KB, 470x454px
>>9077443
"gets them all Right" is statement R
"gets an A" is statement A
for clarity's sake, -R is "gets them all wrong"

The teacher says [math]R\implies A[/math]
This is logically equivalent to [math]\neg A\implies \neg R[/math] because it's the contrapositive

(A) is [math] \neg A \implies -R[/math]
this is wrong because A being false implies R is false, not that the opposite of R is true.
(B) is [math]\neg A \implies \neg R[/math]
This is the contrapositive of [math]R\implies A[/math] so it must be true
(C) is [math] \neg R \implies \neg A[/math]
this would only be true if [math]R \iff A[/math] which isn't necessarily true based on [math]R\implies A[/math]. We don't know if [math]A\implies R[/math]
(D) is [math]A \implies R[/math]
this would be true if [math]R\iff A[/math] was true but we don't know that; that's also why (C) is wrong.
(E) is [math]A \implies \neg -R[/math]
This would only be true if [math]\neg -R \implies R[/math], because [math]R \implies A[/math] and this would simplify to [math]A \implies A[/math] which is a tautology (and therefore true). Unfortunately, [math]\neg -R \not\implies R[/math] Because not getting all the answers wrong doesn't imply that you got them all right.

[math]\not\implies[/math] "does not imply" looks like shit but I did the best I could possibly do under the circumstances.
>>
>>9077472
-R is not "gets them all wrong", its "does not get them all right", i.e. "gets one of them wrong at least".
>>
>>9077450
what the fuck is this?
the problem with the proof is that 4 - 9/2 = -1/2. >>9076924 intentionally written 4.5 as 9/2 to make his 2nd line (with the sqrt of the square) less obviously wrong.
>>
>>9077423
Unless there's only one question.
>>
>>9077458
>Where does it imply that you can't get an A without getting all of the multiple choice right?
That's why E is correct. In some tests you can miss a few questions and still get an A. Hence, if he got an A, the ONLY sentence which necessarily follows is he got AT LEAST one right. It's the only response which covers all possible configurations and makes the statements true.
>>
>>9077509
no, that's [math]\neg R[/math]
-R is "gets them all wrong".
I could have just as easily called it W; it's a completely separate statement.
>>
>>9077552
the problem is that 25 is (-5)^2
If you change 25-45+(9/2)^2 to (9/2 -5)^2 everything works out fine.
>>
>>9077570
When you learn to disconnect world parameters from discrete structures, you'll understand why the answer is B.
>>
>>9077570
He could have received an A even if he got all the questions wrong.
All you know is that if he gets all the questions right he gets an A, but that's not necessarily the only way to get an A. And we don't know the minimum number of correct answers guarantees an A. It could be zero.
>>
>>9077694
B is correct. So is E.
>>
>>9077687
Oh sorry, I thought you meant the negation of R by -R.
>>
>>9077450
sqrt(x^2) = abs(x) =/= x in general and in particular if x = -0.5.
>>
Two trains, A and B, are 150km apart and are moving towards each other at 80km/hr, at t=0 a superhero flying at 120km/hr starting at train A begins to fly back and forth between the trains as they approach one another.

What is the total distance the superhero travels by the time the trains collide?
>>
>>9077724
The trains collide after (75/80)*1 hour, so the superhero travels 0.94*120 = 112 km.
>>
File: 1397121535471.jpg (465KB, 2304x3456px) Image search: [Google]
1397121535471.jpg
465KB, 2304x3456px
>>9077711
E is not correct.

Consider the following scenario: The teacher gives everyone A, regardless of how they answer in the test. In this case you can't deduce that Lewis got at least one question right, since he could get A even if he answered wrong in all the questions.
>>
File: galois.jpg (20KB, 440x600px) Image search: [Google]
galois.jpg
20KB, 440x600px
Let [math] n [/math] be an odd integer [math] >2 [/math] and let [math] f(x)\in \mathbb{Q}[x] [/math] be an irreducible polynomial of degree [math] n [/math] such that the Galois group [math] Gal(f/\mathbb{Q}) [/math] is isomorphic to the dihedral group [math] D_n [/math] of order [math] 2n [/math]. Let [math] \alpha [/math] be a real root of [math] f(x) [/math]. Prove [math] \alpha [/math] can be expressed by real radicals if and only if every prime divisor of [math] n [/math] is a Fermat prime.
>>
>>9077976
you could use that exact same scenario with B.
>>
>>9078147
are you dense? if the teacher gives everyone an A then B can obviously not be true
>>
>>9076892
trivial
>>
File: wrong.png (82KB, 332x498px) Image search: [Google]
wrong.png
82KB, 332x498px
>>9078299

Looks like someone needs to Get Rational With These 4 Weird Tricks

https://www.dropbox.com/sh/8sf1tvfgg8ujn5j/AABjtx9Za-qFuj60imWns26sa?dl=0&preview=LogicBook-Draft+of+7-16-17.pdf
>>
>>9077408
>>9076899
brainlets.

The question doesn't say that the multiple choice section is the only way to get an A. maybe Lewis wrote a killer essay and she gave him a ton of extra credit. maybe Lewis is banging Ms Carroll after class so he gets an A no matter what. who knows. the point is E doesn't follow.

B is objectively correct, and the proof is here >>9077423
Put in concrete terms, Ms Carroll essentially promised that if you got every multiple choice question correct, she'll ignore the rest of the test and just give you an instant A. Thus if Lewis did not get an A, then he did not get the "instant A", which means he did not get all the multiple choice right.
>>
>>9078147
Kek
If the teacher gives everyone an A unconditionally, then "if you didn't get an A, you got at least one question wrong" is still obviously true
>>
>>9078329
t. Josh Dever
>>
File: 1473792820457.jpg (54KB, 566x480px) Image search: [Google]
1473792820457.jpg
54KB, 566x480px
>>9080228
lmao brainlet
>>
>>9076883
I know B is correct, but why couldn't it be D as well?
>>
>>9080254
Stupid, dumb, brainlet scum
https://en.m.wikipedia.org/wiki/Vacuous_truth

If nobody did not get an A, then clearly the set of people who didn't get an A is contained within the set of people who got at least one question wrong, because the former is the empty set
>>
File: 1500562989797.jpg (41KB, 375x300px) Image search: [Google]
1500562989797.jpg
41KB, 375x300px
>>9077724
Consider instead of the original question that the superhero travels linearly until the point in time at which the two trains collide. Then the distance that the superhero travels is dependent only on the final tIme t.

We have that the trains move at the same velocity; meaning that they will meet after traveling equal distances. Thus 2v=2x/t implies that t=x/v. Inputting, t=15/16 hours.

Hence the superhero travels a total distance x=120×15/16=30×15/4=450/4 miles=112.5 miles
>>
>>9080303
D doesn't follow necessarily but it may be true, but not necessarily true. Lewis maybe aced all other questions but the multiple choice questions and thus received an A.
>>
>>9077443
Consider a test with 10 questions out of which 5 are multiple choice questions. Acing the other five questions could be enough for Lewis to get an A. So a potential scenario is that Lewis gets an A without getting any if the multiple choice questions right. Hence it doesn't follow necessarily that Lewis got one of the multiple choice questions right.
>>
>>9081189
Ah, I see. I thought the whole exam was multiple choice questions. Tricky.
Thread posts: 58
Thread images: 15


[Boards: 3 / a / aco / adv / an / asp / b / bant / biz / c / can / cgl / ck / cm / co / cock / d / diy / e / fa / fap / fit / fitlit / g / gd / gif / h / hc / his / hm / hr / i / ic / int / jp / k / lgbt / lit / m / mlp / mlpol / mo / mtv / mu / n / news / o / out / outsoc / p / po / pol / qa / qst / r / r9k / s / s4s / sci / soc / sp / spa / t / tg / toy / trash / trv / tv / u / v / vg / vint / vip / vp / vr / w / wg / wsg / wsr / x / y] [Search | Top | Home]

I'm aware that Imgur.com will stop allowing adult images since 15th of May. I'm taking actions to backup as much data as possible.
Read more on this topic here - https://archived.moe/talk/thread/1694/


If you need a post removed click on it's [Report] button and follow the instruction.
DMCA Content Takedown via dmca.com
All images are hosted on imgur.com.
If you like this website please support us by donating with Bitcoins at 16mKtbZiwW52BLkibtCr8jUg2KVUMTxVQ5
All trademarks and copyrights on this page are owned by their respective parties.
Images uploaded are the responsibility of the Poster. Comments are owned by the Poster.
This is a 4chan archive - all of the content originated from that site.
This means that RandomArchive shows their content, archived.
If you need information for a Poster - contact them.